4

Vandermondeの行列式の派生形

408
0
$$$$

この記事ではVandermondeの行列式の派生形を紹介します.

Vandermondeの行列式

$$\begin{vmatrix} 1&1&\cdots&1\\ x_1&x_2&\cdots&x_n\\ \vdots&\vdots&\ddots&\vdots\\ {x_1}^{n-1}&{x_2}^{n-1}&\cdots&{x_n}^{n-1} \end{vmatrix}=\prod_{1\leq i< j\leq n}(x_j-x_i)$$

$i\neq j$かつ$x_i=x_j$のとき左辺の行列式は$0$となるので, 左辺を$x_1,\ldots,x_n$の多項式として見たときに$x_i-x_j$で割りきれることがわかる.
したがって, ある多項式$P(x_1,\ldots,x_n)$を用いて
$$\begin{vmatrix} 1&1&\cdots&1\\ x_1&x_2&\cdots&x_n\\ \vdots&\vdots&\ddots&\vdots\\ {x_1}^{n-1}&{x_2}^{n-1}&\cdots&{x_n}^{n-1} \end{vmatrix} =P(x_1,\ldots,x_n)\prod_{1\leq i< j\leq n}(x_j-x_i)$$
と書ける.
両辺の次数を比較すると$P$$0$次であることがわかる.
両辺の$x_2{x_3}^2\cdots{x_n}^{n-1}$の係数を比較すると$P=1$が得られ, 定理が示された.

以下, $\displaystyle V_n\coloneqq\prod_{1\leq i< j\leq n}(x_j-x_i)$とする.

$$\begin{vmatrix} 1&1&\cdots&1\\ x_1&x_2&\cdots&x_n\\ \vdots&\vdots&\ddots&\vdots\\ {x_1}^{n-2}&{x_2}^{n-2}&\cdots&{x_n}^{n-2}\\ {x_1}^{n}&{x_2}^{n}&\cdots&{x_n}^{n} \end{vmatrix}=(x_1+\cdots+x_n)V_n$$

定理1の証明と同様にして, $1$次多項式$P$を用いて
$$\begin{vmatrix} 1&1&\cdots&1\\ x_1&x_2&\cdots&x_n\\ \vdots&\vdots&\ddots&\vdots\\ {x_1}^{n-2}&{x_2}^{n-2}&\cdots&{x_n}^{n-2}\\ {x_1}^{n}&{x_2}^{n}&\cdots&{x_n}^{n} \end{vmatrix} =P(x_1,\ldots,x_n)V_n$$
と書けることがわかる.
ここで, 左辺は交代式であるため, $P$は対称式である.
また, 左辺および$V_n$は斉次式であるため, $P$は斉次式である.

$P$は斉次な$1$次の対称式なので定数$C$を用いて$P=C(x_1+\cdots+x_n)$と書ける.
両辺の$x_2{x_3}^2\cdots{x_{n-1}}^{n-2}{x_n}^n$の係数を比較すると$C=1$が得られるので定理が示された.

$k\geq 0$のとき,
$$\begin{vmatrix} 1&1&\cdots&1\\ x_1&x_2&\cdots&x_n\\ \vdots&\vdots&\ddots&\vdots\\ {x_1}^{n-2}&{x_2}^{n-2}&\cdots&{x_n}^{n-2}\\ {x_1}^{n+k-1}&{x_2}^{n+k-1}&\cdots&{x_n}^{n+k-1} \end{vmatrix} =\sum_{1\leq i_1\leq\cdots\leq i_k\leq n}x_{i_1}\cdots x_{i_k}V_n$$

$(n,k)$に関する帰納法による.
$$\begin{aligned} &\begin{vmatrix} 1&1&\cdots&1\\ x_1&x_2&\cdots&x_n\\ \vdots&\vdots&\ddots&\vdots\\ {x_1}^{n-2}&{x_2}^{n-2}&\cdots&{x_n}^{n-2}\\ {x_1}^{n+k-1}&{x_2}^{n+k-1}&\cdots&{x_n}^{n+k-1} \end{vmatrix}\\ &=\begin{vmatrix} 1&0&\cdots&0\\ x_1&x_2-x_1&\cdots&x_n-x_1\\ \vdots&\vdots&\ddots&\vdots\\ {x_1}^{n-2}&{x_2}^{n-2}-{x_1}^{n-1}&\cdots&{x_n}^{n-2}-{x_1}^{n-1}\\ {x_1}^{n+k-1}&{x_2}^{n+k-1}-{x_1}^{n+k-1}&\cdots&{x_n}^{n+k-1}-{x_1}^{n+k-1} \end{vmatrix}\\ &=\prod_{i=2}^{n}(x_i-x_1)\times\begin{vmatrix} 1&\cdots&1\\ \vdots&\ddots&\vdots\\ {x_2}^{n-3}+\cdots+{x_1}^{n-3}&\cdots&{x_n}^{n-3}+\cdots+{x_1}^{n-3}\\ {x_2}^{n+k-2}+\cdots+{x_1}^{n+k-2}&\cdots&{x_n}^{n+k-2}+\cdots+{x_1}^{n+k-2} \end{vmatrix}\\ &=\prod_{i=2}^{n}(x_i-x_1)\times\begin{vmatrix} 1&\cdots&1\\ \vdots&\ddots&\vdots\\ {x_2}^{n-3}&\cdots&{x_n}^{n-3}\\ {x_2}^{n+k-2}+\cdots+{x_2}^{n-2}{x_1}^{k}&\cdots&{x_n}^{n+k-2}+\cdots+{x_1}^{n+k-2} \end{vmatrix} \end{aligned}$$
であるため, $(n-1,0),\ldots,(n-1,k)$で成り立てば$(n,k)$で成り立つ.
$n=1$のとき成り立っているため, 任意の$(n,k)$で成り立つ.

$0\leq k\leq n$のとき,
$$\begin{vmatrix} 1&1&\cdots&1\\ x_1&x_2&\cdots&x_n\\ \vdots&\vdots&\ddots&\vdots\\ {x_1}^{n-k-1}&{x_2}^{n-k-1}&\cdots&{x_n}^{n-k-1}\\ {x_1}^{n-k+1}&{x_2}^{n-k+1}&\cdots&{x_n}^{n-k+1}\\ \vdots&\vdots&\ddots&\vdots\\ {x_1}^{n}&{x_2}^{n}&\cdots&{x_n}^{n} \end{vmatrix} =S_k(x_1,\ldots,x_n)V_n$$
ここで, $S_k$$k$次の基本対称式である.

$$\begin{vmatrix} 1&1&\cdots&1&1\\ x_1&x_2&\cdots&x_n&z\\ \vdots&\vdots&\ddots&\vdots&\vdots\\ {x_1}^{n-k-1}&{x_2}^{n-k-1}&\cdots&{x_n}^{n-k-1}&z^{n-k-1}\\ {x_1}^{n-k}&{x_2}^{n-k}&\cdots&{x_n}^{n-k}&z^{n-k}\\ {x_1}^{n-k+1}&{x_2}^{n-k+1}&\cdots&{x_n}^{n-k+1}&z^{n-k+1}\\ \vdots&\vdots&\ddots&\vdots&\vdots\\ {x_1}^{n}&{x_2}^{n}&\cdots&{x_n}^{n}&z^n \end{vmatrix}=\prod_{i=1}^{n}(z-x_i)\cdot V_n$$
である.
左辺を第$n+1$列で余因子展開して$z^{n-k}$の係数を比較すると定理が得られる.

$-1\leq j< k\leq n$のとき,
$$\begin{vmatrix} 1&1&\cdots&1\\ x_1&x_2&\cdots&x_n\\ \vdots&\vdots&\ddots&\vdots\\ {x_1}^{n-k-1}&{x_2}^{n-k-1}&\cdots&{x_n}^{n-k-1}\\ {x_1}^{n-k+1}&{x_2}^{n-k+1}&\cdots&{x_n}^{n-k+1}\\ \vdots&\vdots&\ddots&\vdots\\ {x_1}^{n-j-1}&{x_2}^{n-j-1}&\cdots&{x_n}^{n-j-1}\\ {x_1}^{n-j+1}&{x_2}^{n-j+1}&\cdots&{x_n}^{n-j+1}\\ \vdots&\vdots&\ddots&\vdots\\ {x_1}^{n+1}&{x_2}^{n+1}&\cdots&{x_n}^{n+1} \end{vmatrix} =(S_kS_{j+1}-S_{k+1}S_j)V_n$$

$\displaystyle(z_2-z_1)\prod_{i=1}^n(z_1-x_i)\prod_{i=1}^n(z_2-x_i)$${z_1}^{n-k}{z_2}^{n-j}$の係数の$(-1)^{j+k+1}$倍を計算することで示される.

$-2\leq i< j< k\leq n$のとき,
$$\begin{vmatrix} 1&1&\cdots&1\\ x_1&x_2&\cdots&x_n\\ \vdots&\vdots&\ddots&\vdots\\ {x_1}^{n-k-1}&{x_2}^{n-k-1}&\cdots&{x_n}^{n-k-1}\\ {x_1}^{n-k+1}&{x_2}^{n-k+1}&\cdots&{x_n}^{n-k+1}\\ \vdots&\vdots&\ddots&\vdots\\ {x_1}^{n-j-1}&{x_2}^{n-j-1}&\cdots&{x_n}^{n-j-1}\\ {x_1}^{n-j+1}&{x_2}^{n-j+1}&\cdots&{x_n}^{n-j+1}\\ \vdots&\vdots&\ddots&\vdots\\ {x_1}^{n-i-1}&{x_2}^{n-i-1}&\cdots&{x_n}^{n-i-1}\\ {x_1}^{n-i+1}&{x_2}^{n-i+1}&\cdots&{x_n}^{n-i+1}\\ \vdots&\vdots&\ddots&\vdots\\ {x_1}^{n+2}&{x_2}^{n+2}&\cdots&{x_n}^{n+2} \end{vmatrix} =\begin{multlined} (S_kS_{j+1}S_{i+2}-S_kS_{j+2}S_{i+1}-S_{k+1}S_jS_{i+2}\\ +S_{k+1}S_{j+2}S_i+S_{k+2}S_jS_{i+1}-S_{k+2}S_{j+1}S_i)V_n \end{multlined}$$

投稿日:14日前

この記事を高評価した人

高評価したユーザはいません

この記事に送られたバッジ

バッジはありません。

投稿者

tria_math
tria_math
509
38898
大学2年生

コメント

他の人のコメント

コメントはありません。
読み込み中...
読み込み中